Hyperbolic cosine identity help

Click For Summary
The discussion centers on proving the hyperbolic cosine identity cosh^2(x) = (cosh(2x) - 1)/2. Participants clarify the correct formula for cosh(x) and discuss the steps to derive the identity. One user attempts to square cosh(x) and expresses confusion over the equivalence to the identity. Another user provides a clearer breakdown of the steps leading to the identity, emphasizing the relationship between cosh and sinh functions. The conversation concludes with a valid derivation of the identity, confirming its correctness.
s.perkins
Messages
4
Reaction score
0

Homework Statement



Show that cosh^2(x) = (cosh(2x) - 1)/2

Homework Equations



cosh(x) = (e^x + e^-x)/2

The Attempt at a Solution



I have attempted this multiple times and get the same results every time.
Squaring cosh(x) I get 1/4(e^2x + e^-2x +2), which is i believe 1/4(cosh(2x) +2).

Maybe i just can't see it but how it that equivalent to the identity above??
 
Last edited:
Physics news on Phys.org
wait, so what are you trying to show?

the first is just normal double angle cos and can be shown by looking at Re and I am of (e^(itheta))^2
 
Last edited:
welcome to pf!

hi s.perkins! welcome to pf!:smile:
s.perkins said:
cosh(x) = (e^x - e^-x)/2

erm :redface:

cosh(x) = (e^x plus e^-x)/2 :wink:
 
looks like the correct form has been used in the square, but also don't forget the factor of 2 as well...
cosh(2x)=(e^(x)+e^(-2x))/2
 
Sorry fixed the typo. I did include the 1/2, when its squared you get a 1/4 in front.
 
s.perkins said:
Squaring cosh(x) I get 1/4(e^2x + e^-2x +2), which is i believe 1/4(cosh(2x) +2).

might as well use equals as its a bit clearer what you're trying

here should be

cosh(x)^2
= (e^2x + e^-2x +2)/4
= (2(e^2x + e^-2x)/2 +2)/4
= (2cosh(2x) +2)/4
= (cosh(2x) +1)/2

which is a valid identity, as shown by adding th two below together
cosh(x)^2- sinh(x)^2=1
cosh(x)^2+sinh(x)^2=cosh(2x)
which gives
2cosh(x)^2=cosh(2x)+1
 
Question: A clock's minute hand has length 4 and its hour hand has length 3. What is the distance between the tips at the moment when it is increasing most rapidly?(Putnam Exam Question) Answer: Making assumption that both the hands moves at constant angular velocities, the answer is ## \sqrt{7} .## But don't you think this assumption is somewhat doubtful and wrong?

Similar threads

  • · Replies 3 ·
Replies
3
Views
1K
  • · Replies 4 ·
Replies
4
Views
1K
  • · Replies 4 ·
Replies
4
Views
1K
  • · Replies 1 ·
Replies
1
Views
1K
  • · Replies 18 ·
Replies
18
Views
3K
Replies
2
Views
1K
  • · Replies 2 ·
Replies
2
Views
2K
Replies
4
Views
2K
  • · Replies 3 ·
Replies
3
Views
2K
Replies
3
Views
1K